沒人做這個?

來源: 康MM 2009-01-13 11:26:26 [] [博客] [舊帖] [給我悄悄話] 本文已被閱讀: 次 (100 bytes)
設第2^n位和第2^m位都是循環節的第k位。則第2^(n+1)位和第2^(m+1)位都是循環節的第2k mod (循環節長度)位。
請您先登陸,再發跟帖!

發現Adblock插件

如要繼續瀏覽
請支持本站 請務必在本站關閉/移除任何Adblock

關閉Adblock後 請點擊

請參考如何關閉Adblock/Adblock plus

安裝Adblock plus用戶請點擊瀏覽器圖標
選擇“Disable on www.wenxuecity.com”

安裝Adblock用戶請點擊圖標
選擇“don't run on pages on this domain”